Những câu hỏi liên quan
Nam thần châu Á
Xem chi tiết
Thiên An
1 tháng 8 2018 lúc 20:41

Ta có bđt \(ab^2+bc^2+ca^2\le\frac{1}{3}\left(a+b+c\right)\left(a^2+b^2+c^2\right)=a^2+b^2+c^2\)

\(P=2017\left(\frac{a^3}{1+b^2}+\frac{b^3}{1+c^2}+\frac{c^3}{1+a^2}\right)\)

Ta có: \(\frac{a^3}{1+b^2}+\frac{a\left(1+b^2\right)}{4}\ge2\sqrt{\frac{a^3}{1+b^2}.\frac{a\left(1+b^2\right)}{4}}=a^2\)

Tương tự suy ra \(\frac{a^3}{1+b^2}+\frac{b^3}{1+c^2}+\frac{c^3}{1+a^2}\ge\left(a^2+b^2+c^2\right)-\frac{1}{4}\left(a+b+c\right)-\frac{1}{4}\left(ab^2+bc^2+ca^2\right)\)

\(\ge\left(a^2+b^2+c^2\right)-\frac{3}{4}-\frac{1}{4}\left(a^2+b^2+c^2\right)=\frac{3}{4}\left(a^2+b^2+c^2\right)-\frac{3}{4}\ge\frac{3}{4}.3-\frac{3}{4}=\frac{3}{2}\)

Bình luận (0)
Kan
Xem chi tiết
KhangCVn
17 tháng 9 2021 lúc 7:18

Ta có :\(\left(a+b+c\right)^2\ge3\left(ab+bc+ac\right)=3\)=> \(a+b+c\ge\sqrt{3}\)

\(\frac{a^3}{b^2+1}=\frac{a^3}{b^2+ab+bc+ac}=\frac{a^3}{\left(b+c\right)\left(b+a\right)}\)

Áp dụng bđt cosi ta có:

\(\frac{a^3}{\left(b+a\right)\left(b+c\right)}+\frac{b+a}{8}+\frac{b+c}{8}\ge3\sqrt[3]{\frac{a^3}{8.8}}=\frac{3}{4}a\)

CM tuong tự

=> \(P+2.\left(\frac{b+a}{8}+\frac{b+c}{8}+\frac{a+c}{8}\right)\ge\frac{3}{4}a+\frac{3}{4}b+\frac{3}{4}c\)

=>\(P\ge\frac{a+b+c}{4}\ge\frac{\sqrt{3}}{4}\)

=>\(MinP=\frac{\sqrt{3}}{4}\)xảy ra khi \(a=b=c=\frac{\sqrt{3}}{3}\)

Bình luận (0)
 Khách vãng lai đã xóa
Diệp Nguyễn Thị Huyền
Xem chi tiết
Khanh Nguyễn Ngọc
19 tháng 7 2021 lúc 21:20

\(P=\frac{a^2}{b^3}+\frac{b^2}{c^3}+\frac{c^2}{a^3}+2-2=\frac{a^2}{b^3}+\frac{b^2}{c^3}+\frac{c^2}{a^3}+2\left(\frac{1}{a}+\frac{1}{b}+\frac{1}{c}\right)-2\)

\(=\left(\frac{a^2}{b^3}+\frac{1}{a}+\frac{1}{a}\right)+\left(\frac{b^2}{c^3}+\frac{1}{b}+\frac{1}{b}\right)+\left(\frac{c^2}{a^3}+\frac{1}{c}+\frac{1}{c}\right)-2\)

Áp dụng BĐT AM-GM cho 3 số dương: 

\(\frac{a^2}{b^3}+\frac{1}{a}+\frac{1}{a}\ge3\sqrt[3]{\frac{a^2}{b^3}.\frac{1}{a}.\frac{1}{a}}=\frac{3}{b}\)

\(\frac{b^2}{c^3}+\frac{1}{b}+\frac{1}{b}\ge3\sqrt[3]{\frac{b^2}{c^3}.\frac{1}{b}.\frac{1}{b}}=\frac{3}{c}\)

\(\frac{c^2}{a^3}+\frac{1}{c}+\frac{1}{c}\ge3\sqrt[3]{\frac{c^2}{a^3}.\frac{1}{c}.\frac{1}{c}}=\frac{3}{a}\)

\(\Rightarrow P\ge\frac{3}{b}+\frac{3}{c}+\frac{3}{a}-2=3-2=1\)

Dấu "=" xảy ra khi \(a=b=c=3\)

Bình luận (0)
 Khách vãng lai đã xóa
alibaba nguyễn
19 tháng 7 2021 lúc 21:24

Đặt \(\frac{1}{a}=x,\frac{1}{b}=y,\frac{1}{c}=z\) thì

\(\Rightarrow\hept{\begin{cases}x+y+z=1\\P=\frac{y^3}{x^2}+\frac{z^3}{y^2}+\frac{x^3}{z^2}\end{cases}}\)

Ta có:

\(\frac{x^3}{z^2}+z+z\ge3x,\frac{y^3}{x^2}+x+x\ge3y,\frac{z^3}{y^2}+y+y\ge3z\)

\(\Rightarrow\frac{x^3}{z^2}\ge3x-2z,\frac{y^3}{x^2}\ge3y-2x,\frac{z^3}{y^2}\ge3z-2y\)

\(\Rightarrow P\ge3x-2z+3y-2x+3z-2y=x+y+z=1\)

Bình luận (0)
 Khách vãng lai đã xóa
nguyễn quỳnh anh
Xem chi tiết
Lê Hồ Trọng Tín
30 tháng 5 2019 lúc 9:21

Từ giả thiết và BĐT AM-GM suy ra:\(\sqrt[3]{\left(a+1\right)\left(b+1\right)\left(c+1\right)}\)\(\ge\)3

Ta có:

P\(\ge\)\(\frac{2a^3}{3\left(a^2+b^2\right)}\)+\(\frac{2b^3}{3\left(c^2+b^2\right)}\)+\(\frac{2c^3}{3\left(a^2+c^2\right)}\)

=\(\frac{2}{3}\)(\(\frac{a\left(a^2+b^2\right)-ab^2}{\left(a^2+b^2\right)}\)+\(\frac{b\left(c^2+b^2\right)-bc^2}{\left(c^2+b^2\right)}\)+\(\frac{a\left(a^2+c^2\right)-ca^2}{\left(a^2+c^2\right)}\))

=\(\frac{2}{3}\)(a+b+c-\(\frac{ab^2}{\left(a^2+b^2\right)}\)-\(\frac{bc^2}{\left(c^2+b^2\right)}\)-\(\frac{ca^2}{\left(a^2+c^2\right)}\))

\(\ge\)\(\frac{2}{3}\)(a+b+c-\(\frac{a}{2}\)-\(\frac{b}{2}\)-\(\frac{c}{2}\))

=\(\frac{2}{3}\).\(\frac{a+b+c}{2}\)=\(\frac{a+b+c}{3}\)=\(\frac{\left(a+1\right)+\left(b+1\right)+\left(c+1\right)}{3}\)-1

\(\ge\)\(\frac{3\sqrt[3]{\left(a+1\right)\left(b+1\right)\left(c+1\right)}}{3}\)-1\(\ge\)2

Vậy:MinP=2 khi a=b=c=2

Bình luận (0)
Thanh Tùng DZ
30 tháng 5 2019 lúc 14:15

cách này dễ hiểu hơn nè :

Áp dụng BĐT : \(\frac{1}{x}+\frac{1}{y}+\frac{1}{z}\ge\frac{9}{x+y+z}\)

Ta có : \(1\ge\frac{1}{a+1}+\frac{1}{b+1}+\frac{1}{c+1}\ge\frac{9}{a+b+c+3}\)

\(\Leftrightarrow1\ge\frac{9}{a+b+c+3}\)\(\Leftrightarrow a+b+c+3\ge9\)\(\Leftrightarrow a+b+c\ge6\)

\(\frac{a^3}{a^2+ab+b^2}=\frac{a\left(a^2+ab+b^2\right)-ab^2-a^2b}{a^2+ab+b^2}=a-\frac{ab^2+a^2b}{a^2+ab+b^2}\ge a-\frac{ab\left(a+b\right)}{3ab}=a-\frac{a+b}{3}\)

Tương tự : \(\frac{b^3}{b^2+bc+c^2}\ge b-\frac{b+c}{3}\)\(\frac{c^3}{c^2+ac+a^2}\ge c-\frac{a+c}{3}\)

Cộng cả 3 vế , ta được : \(P\ge a+b+c-\frac{2\left(a+b+c\right)}{3}=\frac{1}{3}\left(a+b+c\right)\ge\frac{1}{3}.6=2\)

Vậy GTNN của P là 2 \(\Leftrightarrow a=b=c=2\)

Bình luận (0)
nguyễn quỳnh anh
30 tháng 5 2019 lúc 21:21

cảm ơn 2 bạn nhìu

Bình luận (0)
Le Trang Nhung
Xem chi tiết
Trần Hữu Ngọc Minh
7 tháng 12 2017 lúc 16:56

bài 1

ÁP dụng AM-GM ta có:

\(\frac{a^3}{b\left(2c+a\right)}+\frac{2c+a}{9}+\frac{b}{3}\ge3\sqrt[3]{\frac{a^3.\left(2c+a\right).b}{b\left(2c+a\right).27}}=a.\)

tương tự ta có:\(\frac{b^3}{c\left(2a+b\right)}+\frac{2a+b}{9}+\frac{c}{3}\ge b,\frac{c^3}{a\left(2b+c\right)}+\frac{2b+c}{9}+\frac{a}{3}\ge c\)

công tất cả lại ta có:

\(P+\frac{2a+b}{9}+\frac{2b+c}{9}+\frac{2c+a}{9}+\frac{a+b+c}{3}\ge a+b+c\)

\(P+\frac{2\left(a+b+c\right)}{3}\ge a+b+c\)

Thay \(a+b+c=3\)vào ta được":

\(P+2\ge3\Leftrightarrow P\ge1\)

Vậy Min là \(1\)

dấu \(=\)xảy ra khi \(a=b=c=1\)

Bình luận (0)
Nguyên Hoàng Quynh Anh
Xem chi tiết
Thanh Tùng DZ
5 tháng 2 2020 lúc 16:49

Áp dụng BĐT Cô-si cho 3 số dương, ta có :

\(\frac{1}{a\left(a+b\right)}+\frac{1}{b\left(b+c\right)}+\frac{1}{c\left(a+c\right)}\ge3\sqrt[3]{\frac{1}{abc\left(a+b\right)\left(b+c\right)\left(a+c\right)}}\)

Cần chứng minh : \(\sqrt[3]{\frac{1}{abc\left(a+b\right)\left(b+c\right)\left(a+c\right)}}\ge\frac{9}{2\left(a+b+c\right)^2}\)

hay \(8\left(a+b+c\right)^6\ge729abc\left(a+b\right)\left(b+c\right)\left(a+c\right)\)

Thật vậy, ta có : \(\left(a+b+c\right)^3\ge\left(3\sqrt[3]{abc}\right)^3=27abc\)

\(8\left(a+b+c\right)^3=\left(2\left(a+b+c\right)\right)^3=\left(a+b+b+c+a+c\right)^3\)

\(\ge\left(3\sqrt[3]{\left(a+b\right)\left(b+c\right)\left(a+c\right)}\right)^3=27\left(a+b\right)\left(b+c\right)\left(a+c\right)\)

Nhân từng vế 2 bất đẳng thức trên, ta được đpcm

Dấu "=" xảy ra khi a = b = c 

Vậy ...

Bình luận (0)
 Khách vãng lai đã xóa
Thanh Tùng DZ
5 tháng 2 2020 lúc 17:01

2. Áp dụng BĐT Cô-si cho 3 số không âm, ta có : 

\(B\ge3\sqrt[3]{\sqrt{\left(a^3+b^3+1\right)\left(b^3+c^3+1\right)\left(a^3+c^3+1\right)}}\)

Ta có : \(a^3+b^3+1\ge3\sqrt[3]{a^3b^3}=3ab\Rightarrow\sqrt{a^3+b^3+1}\ge\sqrt{3ab}\)

Tương tự : ....

\(\Rightarrow\sqrt{\left(a^3+b^3+1\right)\left(b^3+c^3+1\right)\left(c^3+a^3+1\right)}\ge\sqrt{27a^2b^2c^2}=\sqrt{27}\)

\(\Rightarrow B\ge3\sqrt[3]{\sqrt{27}}=3\sqrt{3}\)

Vậy GTNN của B là \(3\sqrt{3}\)khi a = b = c = 1

Bình luận (0)
 Khách vãng lai đã xóa
Nguyễn Thị Mát
5 tháng 2 2020 lúc 17:06

Bài 1 : 

Áp dụng bất đẳng thức Cauchy cho 3 bộ số thực không âm 

\(\Rightarrow\frac{1}{a\left(a+b\right)}+\frac{1}{b\left(b+c\right)}+\frac{1}{c\left(c+a\right)}\ge3\sqrt[3]{\frac{1}{abc\left(a+b\right)\left(b+c\right)\left(c+a\right)}}\)

\(\Rightarrow\frac{1}{a\left(a+b\right)}+\frac{1}{b\left(b+c\right)}+\frac{1}{c\left(c+a\right)}\ge\frac{3}{\sqrt[3]{abc\left(a+b\right)\left(b+c\right)\left(c+a\right)}}\)

Xét \(\frac{3}{\sqrt[3]{abc\left(a+b\right)\left(b+c\right)\left(c+a\right)}}\)

Áp dụng bất đẳng thức Cauchy cho 3 bộ số thực không âm

\(\hept{\begin{cases}\sqrt[3]{abc}\le\frac{a+b+c}{3}\\\sqrt[3]{\left(a+b\right)\left(b+c\right)\left(c+a\right)}\le\frac{2\left(a+b+c\right)}{3}\end{cases}}\)

Nhân từng vế :

\(\Rightarrow\sqrt[3]{abc\left(a+b\right)\left(b+c\right)\left(c+a\right)}\le\frac{2\left(a+b+c\right)^2}{9}\)

\(\Rightarrow\frac{3}{\sqrt[3]{abc\left(a+b\right)\left(b+c\right)\left(c+a\right)}}\ge\frac{27}{2\left(a+b+c\right)^2}\)

Mà \(\frac{1}{a\left(a+b\right)}+\frac{1}{b\left(b+c\right)}+\frac{1}{c\left(c+a\right)}\ge\frac{3}{\sqrt[3]{abc\left(a+b\right)\left(b+c\right)\left(c+a\right)}}\)

\(\Rightarrow\frac{1}{a\left(a+b\right)}+\frac{1}{b\left(b+c\right)}+\frac{1}{c\left(c+a\right)}\ge\frac{27}{2\left(a+b+c\right)^2}\left(đpcm\right)\)

Bình luận (0)
 Khách vãng lai đã xóa
Edogawa Conan
Xem chi tiết
Kiệt Nguyễn
17 tháng 10 2020 lúc 11:25

2. \(BĐT\Leftrightarrow\frac{1}{1+\frac{2}{a}}+\frac{1}{1+\frac{2}{b}}+\frac{1}{1+\frac{2}{c}}\ge1\)

Đặt\(\frac{2}{a}=x;\frac{2}{b}=y;\frac{2}{c}=z\)thì \(\hept{\begin{cases}x,y,z>0\\xyz=8\end{cases}}\)

Ta cần chứng minh \(\frac{1}{1+x}+\frac{1}{1+y}+\frac{1}{1+z}\ge1\Leftrightarrow\left(yz+y+z+1\right)+\left(zx+z+x+1\right)+\left(xy+x+y+1\right)\ge xyz+\left(xy+yz+zx\right)+\left(x+y+z\right)+1\)\(\Leftrightarrow x+y+z\ge6\)(Đúng vì \(x+y+z\ge3\sqrt[3]{xyz}=6\))

Đẳng thức xảy ra khi x = y = z = 2 hay a = b = c = 1

Bình luận (0)
 Khách vãng lai đã xóa
Kiệt Nguyễn
17 tháng 10 2020 lúc 11:38

3. Ta có: \(a+b+c\le\sqrt{3}\Rightarrow\left(a+b+c\right)^2\le3\)

Ta có đánh giá quen thuộc \(\left(a+b+c\right)^2\ge3\left(ab+bc+ca\right)\)

Từ đó suy ra \(ab+bc+ca\le1\)

\(A=\frac{\sqrt{a^2+1}}{b+c}+\frac{\sqrt{b^2+1}}{c+a}+\frac{\sqrt{c^2+1}}{a+b}\ge\frac{\sqrt{a^2+ab+bc+ca}}{b+c}+\frac{\sqrt{b^2+ab+bc+ca}}{c+a}+\frac{\sqrt{c^2+ab+bc+ca}}{a+b}\)\(=\frac{\sqrt{\left(a+b\right)\left(a+c\right)}}{b+c}+\frac{\sqrt{\left(b+a\right)\left(b+c\right)}}{c+a}+\frac{\sqrt{\left(c+a\right)\left(c+b\right)}}{a+b}\ge3\sqrt[3]{\frac{\left(a+b\right)\left(b+c\right)\left(c+a\right)}{\left(a+b\right)\left(b+c\right)\left(c+a\right)}}=3\)Đẳng thức xảy ra khi \(a=b=c=\frac{1}{\sqrt{3}}\)

Bình luận (0)
 Khách vãng lai đã xóa
hh hh
Xem chi tiết
alibaba nguyễn
27 tháng 3 2017 lúc 20:06

Ta có:

\(\frac{3}{2}\ge a+b+c\ge3\sqrt[3]{abc}\)

\(\Leftrightarrow abc\le\frac{1}{8}\)

Ta có:

\(3+\frac{1}{a}+\frac{1}{b}=1+1+1+\frac{1}{2a}+\frac{1}{2a}+\frac{1}{2b}+\frac{1}{2b}\ge7\sqrt[7]{\frac{1}{16a^2b^2}}\)

Tưng tự ta có: \(\hept{\begin{cases}3+\frac{1}{b}+\frac{1}{c}\ge7\sqrt[7]{\frac{1}{16b^2c^2}}\\3+\frac{1}{c}+\frac{1}{a}\ge7\sqrt[7]{\frac{1}{16c^2a^2}}\end{cases}}\)

Từ đó ta có

P\(\ge7\sqrt[7]{\frac{1}{16a^2b^2}}.7\sqrt[7]{\frac{1}{16b^2c^2}}.7\sqrt[7]{\frac{1}{16c^2a^2}}\)

\(=7^3\sqrt[7]{\frac{1}{16^3a^4b^4c^4}}\ge7^3.\sqrt[7]{\frac{8^4}{16^3}}=7^3\)

Dấu = xảy ra khi \(a=b=c=\frac{1}{2}\)

Bình luận (0)
alibaba nguyễn
27 tháng 3 2017 lúc 20:04

Ta có:

\(\frac{3}{2}\ge a+b+c\ge3\sqrt[3]{abc}\)

\(\Leftrightarrow abc\le\frac{1}{8}\)

Ta có:

\(3+\frac{1}{a}+\frac{1}{b}=1+1+1+\frac{1}{2a}+\frac{1}{2a}+\frac{1}{2b}+\frac{1}{2b}\ge7\sqrt[7]{\frac{1}{16a^2b^2}}\)

Tưng tự ta có: \(\hept{\begin{cases}3+\frac{1}{b}+\frac{1}{c}\ge7\sqrt[7]{\frac{1}{16b^2c^2}}\\3+\frac{1}{c}+\frac{1}{a}\ge7\sqrt[7]{\frac{1}{16c^2a^2}}\end{cases}}\)

Từ đó ta có

\(\ge7\sqrt[7]{\frac{1}{16a^2b^2}}.7\sqrt[7]{\frac{1}{16b^2c^2}}.7\sqrt[7]{\frac{1}{16c^2a^2}}\)

\(=7^3\sqrt[7]{\frac{1}{16^3a^4b^4c^4}}\ge7^3.\sqrt[7]{\frac{8^4}{16^3}}=7^3\)

Dấu bằng xảy ra khi \(a=b=c=\frac{1}{2}\)

Bình luận (0)
Secret
27 tháng 3 2017 lúc 20:48

ý bn là cho biểu thức abc vào trong căn thì nó đổi dấu ạ ?

Bình luận (0)
fan FA
Xem chi tiết